REZA 73

Active Member
ارسال ها
139
لایک ها
184
امتیاز
43
پاسخ : ماراتن نظریه اعداد (سطح مقدماتی)

100 عدد متوالی
رو در نظر بگیرین طبق قضیه ی ویلسون میدونیم که
بر 101 بخشپذیره پس از بین اعداد 2 تا 101 هر کدومو انتخاب کرده باشیم این عدد حداقل یکی از اعداد ذکر شده رو میشماره.

---- دو نوشته به هم متصل شده است ----

یه کم سطح سوالا رو بالا می برم. سوال بعد:
ثابت کنید هیچ سه تایی
از اعداد طبیعی وجود ندارد که داشته باشیم:

الان اگه سه رو انتخاب کنیم و از بین اون اعدادی که گفتید 100!+5 رو انتخاب کنیم چطوری عاد میکنه??? اعداد به تصادف انتخاب میشن ها .
 

Dadgarnia

New Member
ارسال ها
1,350
لایک ها
1,127
امتیاز
0
پاسخ : ماراتن نظریه اعداد (سطح مقدماتی)

الان اگه سه رو انتخاب کنیم و از بین اون اعدادی که گفتید 100!+5 رو انتخاب کنیم چطوری عاد میکنه??? اعداد به تصادف انتخاب میشن ها .
بله حق با شماست جوب زدم :4:.
 

Dadgarnia

New Member
ارسال ها
1,350
لایک ها
1,127
امتیاز
0
پاسخ : ماراتن نظریه اعداد (سطح مقدماتی)

اين سوال ايده اش باقي مانده چينيه نه؟ من تا حدودي تونستم ثابت كنم ولي هنوز كامل ثابت نشده. ميشه ثابت كرد هميشه بي نهايت عدد دو به دو نسبت به هم اول توي اين دنباله وجود داره؟
 

REZA 73

Active Member
ارسال ها
139
لایک ها
184
امتیاز
43
پاسخ : ماراتن نظریه اعداد (سطح مقدماتی)

اين سوال ايده اش باقي مانده چينيه نه؟ من تا حدودي تونستم ثابت كنم ولي هنوز كامل ثابت نشده. ميشه ثابت كرد هميشه بي نهايت عدد دو به دو نسبت به هم اول توي اين دنباله وجود داره؟
ابتدا در مورد سوالی که گذاشته بودین:
اگه معادله رو بر حسب z مرتب كنيم داریم :
یا
حالا ثابت میکنم که هیچ کدوم جواب نداره مثلا دومی اون یکی مشابهه:


حالا فرض کنید تمام عوامل مشترک d و z رو ساده کرده باشیم اونوقت باید داشته باشیم
abو a-b نسبت به هم اولن پس:



و این تناقضه .میشه از فرم اعداد فیثاغورسی برای رسید به تناقض استفاده کرد. البته یه حالت دیگه هم وجود داره گه:

که این هم مشابه قبلیه فقط تو این حالت m,d نسبت به اولا.



در مورد باقیمانده چینی هم نمیدونم حل میشه یا نه ولی راه حل ساده تری داره

---- دو نوشته به هم متصل شده است ----

ابتدا در مورد سوالی که گذاشته بودین:
اگه معادله رو بر حسب z مرتب كنيم داریم :
یا
حالا ثابت میکنم که هیچ کدوم جواب نداره مثلا دومی اون یکی مشابهه:


حالا فرض کنید تمام عوامل مشترک d و z رو ساده کرده باشیم اونوقت باید داشته باشیم
abو a-b نسبت به هم اولن پس:



و این تناقضه .میشه از فرم اعداد فیثاغورسی برای رسید به تناقض استفاده کرد. البته یه حالت دیگه هم وجود داره گه:

که این هم مشابه قبلیه فقط تو این حالت m,d نسبت به اولا.



در مورد باقیمانده چینی هم نمیدونم حل میشه یا نه ولی راه حل ساده تری داره
ادامه ی اثبات:اگه a-b رو در a+b ضرب کنیم به این میرسیم که s به توان 4 منهای t به توان 4 یه مربع کامل میشه که این معادله هم جواب نداره، فک کنم معادله معروفیه.
:178::178:
 
آخرین ویرایش توسط مدیر

محمد 628

New Member
ارسال ها
169
لایک ها
42
امتیاز
0
پاسخ : ماراتن نظریه اعداد (سطح مقدماتی)

سوال بعد
کوچکترین n>100 ای رو پیدا کنین که
بر n بخشپذیر باشه.
 
آخرین ویرایش توسط مدیر

m-saghaei

New Member
ارسال ها
338
لایک ها
258
امتیاز
0
پاسخ : ماراتن نظریه اعداد (سطح مقدماتی)

سوال بعد
کوچکترین n>100 ای رو پیدا کنین که
بر n بخشپذیر باشه.
لطفا اگه کسی جواب این سوالو میدونه بگه تا بریم سوال بعدی!
تقریبا 20 روز گذشته!
 

Dadgarnia

New Member
ارسال ها
1,350
لایک ها
1,127
امتیاز
0
پاسخ : ماراتن نظریه اعداد (سطح مقدماتی)

این سوال قشنگیه فک کنم یه بار تو همین تاپیک گذاشتم ولی کسی چیزی پست نکرد:
از بین هر 100 عدد متوالی طبیعی یک عدد را انتخاب و در مجموعه A میریزیم،ثابت کنید دو تا عدد مثل a و b در A هست که a عاد میکنه b رو.
يه راهنمايي براي اين سوال مي كنيد؟

---- دو نوشته به هم متصل شده است ----

لطفا اگه کسی جواب این سوالو میدونه بگه تا بریم سوال بعدی!
تقریبا 20 روز گذشته!
اين سوال فكر نكنم راه ديگه اي به غير از استفاده از كامپيوتر داشته باشه البته بد نيست كه اين لينكو ببينين: AoPS Forum - A 74 • Art of Problem Solving يه سوال مشابه ست. من كه ايده ي اين كسي كه جوابو نوشته نفهميدم ولي شايد بتونه به حل اين سوال كمك كنه.
 

REZA 73

Active Member
ارسال ها
139
لایک ها
184
امتیاز
43
پاسخ : ماراتن نظریه اعداد (سطح مقدماتی)

راهنمایی:


ثابت کنید یه b وجود داره که به ازای
داریم
 

Dadgarnia

New Member
ارسال ها
1,350
لایک ها
1,127
امتیاز
0
پاسخ : ماراتن نظریه اعداد (سطح مقدماتی)

راهنمایی:


ثابت کنید یه b وجود داره که به ازای
داریم
خودم هم اين به ذهنم رسيده بود ولي نمي دونم چرا اون وقت حل نشد.
ميايم دنباله ي
رو اينجوري تعريف مي كنيم
و
هم يه عدد دلخواه و طبيعيه. واضحه كه
براي هر
و
به راحتي مي تونيم ببينيم كه توي دنباله هاي
به هر صورتي عدد ها رو انتخاب كنيم دو عدد a,b پيدا ميشن كه a ميشمره b رو.

---- دو نوشته به هم متصل شده است ----

سوال بعد:
تمام جفت اعداد طبيعي
را بيابيد به طوريكه
مربع يك عدد اول باشد.
 

REZA 73

Active Member
ارسال ها
139
لایک ها
184
امتیاز
43
پاسخ : ماراتن نظریه اعداد (سطح مقدماتی)

خودم هم اين به ذهنم رسيده بود ولي نمي دونم چرا اون وقت حل نشد.
ميايم دنباله ي
رو اينجوري تعريف مي كنيم
و
هم يه عدد دلخواه و طبيعيه. واضحه كه
براي هر
و
به راحتي مي تونيم ببينيم كه توي دنباله هاي
به هر صورتي عدد ها رو انتخاب كنيم دو عدد a,b پيدا ميشن كه a ميشمره b رو.

---- دو نوشته به هم متصل شده است ----

سوال بعد:
تمام جفت اعداد طبيعي
را بيابيد به طوريكه
مربع يك عدد اول باشد.
این طوری حل میشه:
حالا در هر حالت x جواب ها به دست می آید. مثلا x=1 :
برای این کافیه y رو زوج و فرد کنید.
یا x=p:
 

AHZolfaghari

Well-Known Member
ارسال ها
935
لایک ها
1,654
امتیاز
93
پاسخ : ماراتن نظریه اعداد (سطح مقدماتی)

سوال بعد
تمام اعداد طبیعی ,C , A,B و اول P را بیابید بطوری که



---- دو نوشته به هم متصل شده است ----

ظاهرا سوال تکراری بود . تو صفحات قبلی همین تاپیک وجود داره.
تمام n های طبیعی رو بیابید که m طبیعی وجود داشته باشد به نحوی که



---- دو نوشته به هم متصل شده است ----

اینم تکراری بود اما این انصافا تکراری نیست !!!!

a , C سه عدد طبیعی و B یک عدد صحیح هستند . اثبات کنید معادله هم نهشتی زیر جواب دارد

 

aras2213

New Member
ارسال ها
216
لایک ها
228
امتیاز
0
پاسخ : ماراتن نظریه اعداد (سطح مقدماتی)

سوال بعد
تمام اعداد طبیعی ,C , A,B و اول P را بیابید بطوری که



---- دو نوشته به هم متصل شده است ----

ظاهرا سوال تکراری بود . تو صفحات قبلی همین تاپیک وجود داره.
تمام n های طبیعی رو بیابید که m طبیعی وجود داشته باشد به نحوی که



---- دو نوشته به هم متصل شده است ----

اینم تکراری بود اما این انصافا تکراری نیست !!!!

a , C سه عدد طبیعی و B یک عدد صحیح هستند . اثبات کنید معادله هم نهشتی زیر جواب دارد

http://www.irysc.com/forum/t9771/
 

AHZolfaghari

Well-Known Member
ارسال ها
935
لایک ها
1,654
امتیاز
93

Dadgarnia

New Member
ارسال ها
1,350
لایک ها
1,127
امتیاز
0
پاسخ : ماراتن نظریه اعداد (سطح مقدماتی)

با اجازه ي دوستان من يه سوال خوب گير آوردم گفتم بيام اينجا بذارم:
همه ي اعداد اول
را بيابيد به طوريكه
براي هر عدد اول
خالي از مربع باشد.
 

REZA 73

Active Member
ارسال ها
139
لایک ها
184
امتیاز
43
پاسخ : ماراتن نظریه اعداد (سطح مقدماتی)

با اجازه ي دوستان من يه سوال خوب گير آوردم گفتم بيام اينجا بذارم:
همه ي اعداد اول
را بيابيد به طوريكه
براي هر عدد اول
خالي از مربع باشد.
اگه q عدد اولی باشه که :
اونوقت 4 عاد میکنه صورت مساله رو. پس p-4 باید فقط عامل اول کوچتر از 4 داشته باشه. یعنی

برای این اعداد کافیه q رو در نظر بگیریم که q عاد میکنه p-9 رو. در این صورت 9عبارت مورد نظر رو عاد میکنه مگر حالتی که p این طوری باشه:

حالا باید این رو حل کنیم:

واضحه که توان 5 صفره. اگه a بزرگتر از دو باشه با بررسی به هنگ 4 میشه فهمید x زوجه و این تناقضه.(کافیه به هنگ 7 بررسی بشه)
حالتی هم که a برابر 1 باشه با بررسی به هنگ سه تناقض واضحه.
پس جواب مساله حالت های خاص این معادله میشه.
ویرایش:
ادامه حل: البته این حالت میمونه:

یه بار به پیمانه 3 بررسی کنیم :a زوجه.
به پیمانه 5(با توجه به زوج بودن a میتوان فهمید x هم زوجه)

پس جواب ها میشن:2و3و5و7و13


# راه حلمو چک نکردم امیدوارم جوب نزده باشم#
 
آخرین ویرایش توسط مدیر
ارسال ها
169
لایک ها
42
امتیاز
0
پاسخ : ماراتن نظریه اعداد (سطح مقدماتی)

سوالم آسونه فقط وقت گیره.همون بخشپذیری دو به توان n منها 3 بر n رو میگم اگر تا 10 روز دیگه حل نشد جوابشو میزارم.
 

Dadgarnia

New Member
ارسال ها
1,350
لایک ها
1,127
امتیاز
0
پاسخ : ماراتن نظریه اعداد (سطح مقدماتی)

سوال بعد:
عددی اول و
اعدادی طبیعی هستند که
ثابت کنید
.
 

aras2213

New Member
ارسال ها
216
لایک ها
228
امتیاز
0
پاسخ : ماراتن نظریه اعداد (سطح مقدماتی)

سوال بعد:
عددی اول و
اعدادی طبیعی هستند که
ثابت کنید
.
فرض کنید
نمایش n در مبنای p باشد،در این صورت طبق قضیه لوکاس،
.

---- دو نوشته به هم متصل شده است ----

سوال بعد:همه
را بیابید به طوری که
.(برای هر a,b,c صحیح)

---- دو نوشته به هم متصل شده است ----

مثل این که سوال قبل رو گذاشته بودن،این یه سوال دیگه:

همه ی m,n طبیعی را بیابید که
.
 

Dadgarnia

New Member
ارسال ها
1,350
لایک ها
1,127
امتیاز
0
پاسخ : ماراتن نظریه اعداد (سطح مقدماتی)

فرض کنید
نمایش n در مبنای p باشد،در این صورت طبق قضیه لوکاس،
.

---- دو نوشته به هم متصل شده است ----

سوال بعد:همه
را بیابید به طوری که
.(برای هر a,b,c صحیح)

---- دو نوشته به هم متصل شده است ----

مثل این که سوال قبل رو گذاشته بودن،این یه سوال دیگه:

همه ی m,n طبیعی را بیابید که
.
اگه
باشه جواب
بدست میاد حالا فرض می کنیم
باشه دو طرف رو به پیمانه ی چهار در نظر می گیریم:
با جایگذاری این رابطه در صورت سوال داریم:
اول فرض می کنیم
فرد باشه که بدست میاد:
حالا فرض می کنیم
که نتیجه میده:
پس این معادله همین سه جوابو فقط داره.

---- دو نوشته به هم متصل شده است ----

سوال بعد:
معادله ی زیر را در اعداد صحیح نامنفی حل کنید:

 

TheOverlord

New Member
ارسال ها
159
لایک ها
282
امتیاز
0
پاسخ : ماراتن نظریه اعداد (سطح مقدماتی)

ابتدا فرض کنید
0"> .

سپس با بررسی باقیمانده های ممکن به پیمانه 5 داریم:
پس

حال به پیمانه 6 بررسی میکنیم.

با بررسی باقیمانده های ممکن به پیمانه 7 داریم:
و با بررسی باقیمانده های ممکن به پیمانه 13 داریم :

پس
حال اگر به پیمانه 9 بررسی کنیم
و با بررسی به پیمانه 25 داریم
پس بدیهتا
پس

حال فرض کنید یکی از این اعداد 0 باشد. بدیهتا اگر
یعنی یک جواب دیگر
است. حال اگر

پس نهایتا این معادله تنها دو جواب
و
را دارد.

---- دو نوشته به هم متصل شده است ----

ثابت کنید دنباله ای حسابی به طول 11 از اعداد اول مثبت کوچکتر از 20000 وجود ندارد.
 
بالا